SlideShare a Scribd company logo
1 of 109
Download to read offline
Civil Engineering Department


Hydraulics and Hydrology – CE 352
    Prof. Majed Abu-Zreig



        Chapter 3
     Water Flow in Pipes
3.1 Description of A Pipe Flow

• Water pipes in our homes and the distribution
  system
• Pipes carry hydraulic fluid to various components
  of vehicles and machines
• Natural systems of “pipes” that carry blood
  throughout our body and air into and out of our
  lungs.



                                                  2
• Pipe Flow: refers to a full water flow in a closed
  conduits or circular cross section under a certain
  pressure gradient.

• The pipe flow at any cross section can be
  described by:
 cross section (A),
 elevation (h), measured with respect to a horizontal
  reference datum.
 pressure (P), varies from one point to another, for a
  given cross section variation is neglected
 The flow velocity (v), v = Q/A.

                                                          3
Difference between open-channel flow and the pipe flow




            Pipe flow                   Open-channel flow
• The pipe is completely filled      • Water flows without
with the fluid being transported.    completely filling the pipe.

• The main driving force is likely   • Gravity alone is the
to be a pressure gradient along      driving force, the water
the pipe.                            flows down a hill.
                                                                4
Types of Flow
 Steady and Unsteady flow
 The flow parameters such as velocity (v), pressure (P)
 and density (r) of a fluid flow are independent of time
 in a steady flow. In unsteady flow they are independent.

 For a steady flow       v t x ,y ,z
                                   o    o   o
                                                0

 For an unsteady flow    v t x ,y ,z
                                    o   o   o
                                                0

 If the variations in any fluid’s parameters are small, the
 average is constant, then the fluid is considered to be
 steady
                                                              5
 Uniform and non-uniform flow
 A flow is uniform if the flow characteristics at any given
 instant remain the same at different points in the
 direction of flow, otherwise it is termed as non-uniform
 flow.

 For a uniform flow    v s t   o
                                       0

 For a non-uniform flow     v s t   o
                                            0




                                                              6
Examples:
   The flow through a long uniform pipe diameter at a constant rate is
    steady uniform flow.


   The flow through a long uniform pipe diameter at a varying rate is
    unsteady uniform flow.


   The flow through a diverging pipe diameter at a constant rate is a
    steady non-uniform flow.


   The flow through a diverging pipe diameter at a varying rate is an
    unsteady non-uniform flow.



                                                                          7
 Laminar and turbulent flow
Laminar flow:
The fluid particles move along smooth well defined path or streamlines
that are parallel, thus particles move in laminas or layers, smoothly
gliding over each other.

Turbulent flow:
The fluid particles do not move in orderly manner and they occupy different
relative positions in successive cross-sections.
There is a small fluctuation in magnitude and direction of the velocity of the
fluid particles

transitional flow
The flow occurs between laminar and turbulent flow
                                                                           8
3.2 Reynolds Experiment
Reynolds performed a very carefully prepared pipe flow
experiment.




                                                         9
Increasing
    flow
  velocity




             10
Reynolds Experiment
• Reynold found that transition from laminar to turbulent
  flow in a pipe depends not only on the velocity, but only
  on the pipe diameter and the viscosity of the fluid.

• This relationship between these variables is commonly
  known as Reynolds number (NR)

            VDr     VD
                   Inertial Forces
      NR       
                 Viscous Forces

  It can be shown that the Reynolds number is a measure of
  the ratio of the inertial forces to the viscous forces in the
  flow
                     FI  ma                  FV   A          11
Reynolds number

     rVD VD
NR     
         
 where V:   mean velocity in the pipe [L/T]
       D:   pipe diameter             [L]
       r:   density of flowing fluid [M/L3]
       :   dynamic viscosity [M/LT]
       :   kinematic viscosity       [L2/T]



                                               12
13
It has been found by many experiments that for flows in
circular pipes, the critical Reynolds number is about 2000

Flow laminar when NR < Critical NR
Flow turbulent when NR > Critical NR


The transition from laminar to turbulent flow does not always
happened at NR = 2000 but varies due to experiments
conditions….….this known as transitional range




                                                             14
Laminar Vs. Turbulent flows

Laminar flows characterized       Turbulent flows characterized
by:                               by
   • low velocities                  • high velocities
   • small length scales             • large length scales
   • high kinematic viscosities      • low kinematic viscosities
   • NR < Critical NR                • NR > Critical NR
   • Viscous forces are              • Inertial forces are
    dominant.                          dominant



                                                             15
Example 3.1
40 mm diameter circular pipe carries water at 20oC.
Calculate the largest flow rate (Q) which laminar flow can
be expected.


  D  0.04m

   1106 at T  20o C
         VD                V (0.04)
  NR          2000                2000  V  0.05m / sec
                          110  6



                       
  Q  V . A  0.05        (0.04) 2  6.28 105 m3 / sec
                       4
                                                               16
3.3 Forces in Pipe Flow
• Cross section and elevation of the pipe are varied along
  the axial direction of the flow.




                                                             17
For Incompressible and Steady flows:

 Conservation law of mass

  r.dVol11'  r.dVol22'  mass flux ( fluid mass)

Mass enters the       Mass leaves the
control volume        control volume

    dVol11'       dVol 2 2'
r.            r.
       dt             dt
       dS1          dS 2
r . A1      r . A2       r . A1.V1  r . A2 .V2  r .Q
       dt            dt
      Continuity equation for
      Incompressible Steady flow              A1.V1  A2 .V2  Q
                                                               18
Apply Newton’s Second Law:
                                   
             dV M V 2 M V1
              
 F  M a  M dt     t

F    x    P A1  P2 A2  Fx  Wx
             1


Fx is the axial direction force exerted on the control volume
by the wall of the pipe.
but M t  r .Q  mass flow rate
 Fx  r.Q(Vx2  Vx1 )                     
                                          F r.Q(V
                                                   
                                                       2
                                                            
                                                           V 1)
F    y    r .Q(V y2  V y1 )
F
                                         Conservation of
      z    r .Q(Vz2  Vz1 )             moment equation
                                                                   19
Example 3.2
dA= 40 mm, dB= 20 mm, PA= 500,000 N/m2, Q=0.01m3/sec.
Determine the reaction force at the hinge.




                                                        20
3.4 Energy Head in Pipe Flow

  Water flow in pipes may contain energy in three
  basic forms:
1- Kinetic energy,
2- potential energy,
3- pressure energy.




                                                    21
Consider the control volume:
•   In time interval dt:
- Water particles at sec.1-1 move to sec. 1`-1` with velocity V1.
- Water particles at sec.2-2 move to sec. 2`-2` with velocity V2.

•     To satisfy continuity equation:
             A1.V1.dt  A2 .V2 .dt
•    The work done by the pressure force
             P . A1.ds1  P . A1.V1.dt
              1            1
                                              ……. on section 1-1

           P2 . A2 .ds2   P2 . A2 .V2 .dt ……. on section 2-2

-ve sign because P2 is in the opposite direction to distance traveled ds2
                                                                            22
• The work done by the gravity force :
                          Work  W .h  mg.h
                           m  r .Volume  A1 L  A1V1dt
• The kinetic energy: Work  rg. A1.V1dt.(h1  h2 )
  1        1       1
    M .V2  M .V1  r. A1.V1.dt (V22  V1 )
         2       2                       2

  2        2       2
The total work done by all forces is equal to the change in
kinetic energy:
                                           1
  P .Q.dt  P2 .Q.dt  rg.Q.dt.(h1  h2 )  r.Q.dt (V22  V1 )
                                                            2
   1
                                           2
 Dividing both sides by rgQdt
    2             2              Bernoulli Equation
  V1 P        V   P
     1  h1  2  2  h2
  2g         2g                Energy per unit weight of water
                                                                   23
                                 OR: Energy Head
Energy head and Head loss in pipe flow

                                         24
2
                V2   P2
           H2          h2
                2g   

Energy =    Kinetic   + Pressure   +
                                       Elevation
head        head          head         head
                      2
               V   P
           H1  1  1  h1
               2g  
Notice that:
• In reality, certain amount of energy loss (hL) occurs when the
water mass flow from one section to another.

• The energy relationship between two sections can be written
as:              2              2
              V1    P        V2     P2
                     h1 
                     1
                                       h2  hL
              2g            2g                           25
Example 3.4




The tank is being drained through 12 in pipe. The discharge = 3200 gpm, The
                                                                              26
Total head loss = 11.5 ft. find the h?
Example
In the figure shown:
Where the discharge through the system is 0.05 m3/s, the total losses through
the pipe is 10 v2/2g where v is the velocity of water in 0.15 m diameter pipe,
the water in the final outlet exposed to atmosphere.
Example
In the figure shown:
Where the discharge through the system is 0.05 m3/s, the total losses through the pipe is 10 v2/2g
where v is the velocity of water in 0.15 m diameter pipe, the water in the final outlet exposed to
atmosphere. Calculate the required height (h =?)
below the tank


    Calculate the required
    height (h =?)
    below the tank
                    0.05
    V   Q
                           2.83m / s
                 4 0.15
         A              2


                  0.05
    V   Q
                          6.366m / s
               4 0.10 
         A             2


    p1 V12        p2 V22
           z1         z 2  hL
    rg 2 g        rg 2 g

    0  0  (h  5)  0 
                          6.3662  20  102.832
                             2 * 9.81        2 * 9.81
    h  21.147 m
Without calculation sketch the (E.G.L) and (H.G.L)
Basic components of a typical pipe
            system




                                     30
Calculation of Head (Energy) Losses:
In General:
When a fluid is flowing through a pipe, the fluid experiences some
 resistance due to which some of energy (head) of fluid is lost.
                          Energy Losses
                          (Head losses)

               Major Losses           Minor losses
        loss of head due to pipe   Loss due to the change of
        friction and to viscous    the velocity of the flowing
        dissipation in flowing     fluid in the magnitude or in
        water                      direction as it moves
                                   through fitting like Valves,
                                   Tees, Bends and Reducers.
                                                                  31
3.5 Losses of Head due to Friction
• Energy loss through friction in the length of pipeline is commonly
  termed the major loss hf
• This is the loss of head due to pipe friction and to the viscous
  dissipation in flowing water.
• Several studies have been found the resistance to flow in a pipe is:

       - Independent of pressure under which the water flows
       - Linearly proportional to the pipe length, L
       - Inversely proportional to some water power of the pipe diameter D
       - Proportional to some power of the mean velocity, V
       - Related to the roughness of the pipe, if the flow is turbulent
The resistance to flow in a pipe is a function of:


•    The pipe length, L
•    The pipe diameter, D
•    The mean velocity, V
•    The properties of the fluid ()
•    The roughness of the pipe, (the flow is
     turbulent).


                                                     33
Darcy-Weisbach Equation
                   2              2              Where:
       L V    8f LQ
hL  f                                         f is the friction factor
       D 2 g g D5  2                            L is pipe length
                                                 D is pipe diameter
                                                 Q is the flow rate
                                                 hL is the loss due to friction
It is conveniently expressed in terms of velocity (kinetic) head in the pipe


The friction factor is function of different terms:

              e     rVD e     VD e 
    f  F NR ,   F    ,   F
                       D          , 
              D                D

  Renold number         Relative roughness
Friction Factor: (f)
• For Laminar flow: (NR < 2000) [depends only on
  Reynolds’ number and not on the surface roughness]
                           64
                       f 
                           NR
 • For turbulent flow in smooth pipes (e/D = 0) with
 4000 < NR < 105 is
                          0.316
                       f  1/ 4
                          NR




                                                       35
Friction Factor f
The thickness of the laminar sublayer  decrease with an increase in NR

laminar flow                                                       f independent of relative
 NR < 2000                                   Smooth                roughness e/D
                 
                                        e    '  1.7e             64       1             N f     
                                                             f                 2 log 10  R
                                                                                           2.51
                                                                                                   
                                                                                                   
                          pipe wall                                NR        f                    
     
                                                                    f varies with NR and e/D

                                                                                   e            
                                            transitionally                                      
                                                                       2 log 10    
                                                rough              1                 D    2.51     
                                        e
                                                                    f              3.7 N R f      
                                                                                                  
                           pipe wall    0.08e   '  1.7e                                        
                                                                   Colebrook formula
      
turbulent flow
                                                                        f independent of NR
                                                 rough
  NR > 4000
                                            e                           1                 D
                                              0.08e
                                                 '                          2 log 10  3.7 
                                                                         f                e
                            pipe wall
Moody diagram
• A convenient chart was prepared by Lewis F. Moody and commonly
  called the Moody diagram of friction factors for pipe flow,
  There are 4 zones of pipe flow in the chart:

• A laminar flow zone where f is simple linear function of NR
• A critical zone (shaded) where values are uncertain because
  the flow might be neither laminar nor truly turbulent
• A transition zone where f is a function of both NR and relative
  roughness
• A zone of fully developed turbulence where the value of f
  depends solely on the relative roughness and independent of
  the Reynolds Number
38
Laminar




          Marks Reynolds Number
          independence
Typical values of the absolute roughness (e) are given in
                        table 3.1




                                                       40
Notes: Alternative to Moody Diagram

• Swamee-Jain Equation (1976)

                       0.25
     f                                 2
                  e       5.74 
           log10  3.7 D  Re 0.9 
                                 
Explicit expression 10-6<e/D,10-2; 5000<NR<108



                                                 41
Problems (head loss)
Three types of problems for uniform flow
in a single pipe:
 Type 1:
Given the kind and size of pipe and the flow rate     head loss ?

 Type 2:
Given the kind and size of pipe and the head loss     flow rate ?


 Type 3:
Given the kind of pipe, the head loss and flow rate   size of pipe ?
Solving Turbulent flow Problems

There are 3 types of problems
• Given: L, D, V solve for hf
   – Compute ks/D, Re then f from moody diagram then
     find hf and V
• Given: hf, L, D solve for V
                                    3/ 2
                                D     2 gh
   – Compute ks/D the value            L
                                           Then calculate f, V
                                            f



     and Q
• Given: Q, L, hf solve for D, Iterative solution
   – Iterative process Assume f, calculate V and Re, check f from
     moody diagram
   – Assume new f, calculate V and Re, then check f
Moody Diagram                                D3 / 2  2 gh f                
                                                                                                                 1/ 2

                                                         N R f 1/ 2                                  
                                                                          L  
                                                                                                         
                                                                                                         




                                 Fully rough pipes
Resistance Coefficient f




                                                                                        Relative roughness e/D
                                Smooth pipes


                                               Reynolds number
Example 1
The water flow in Asphalted cast Iron pipe (e = 0.12mm) has a diameter 20cm
at 20oC. Is 0.05 m3/s. determine the losses due to friction per 1 km
   Type 1:
  Given the kind and size of pipe and the flow rate            head loss ?
         0.05m 3 /s
    V                 1.59m/s
                
       π/4  0.2 m
                 2  2
                          
   T  20o C  υ  1.0110 6 m 2 /s
   e  0.12mm
   e 0.12mm
             0.0006                                    Moody   f = 0.018
   D 200mm
        VD 1.59  0.2
   NR                314852  3.15 105
          1.0110 6
          L V2         1,000 m  1.59              
                                         2
   hf  f       0.018                            
          D 2g                        
                       0.20 m  2 9.81 m/s
                                             2
                                                    
                                                     
                                                                             45
       11.55 m
Example 2
The water flow in commercial steel pipe (e = 0.045mm) has a diameter 0.5m
at 20oC. Q=0.4 m3/s. determine the losses due to friction per 1 km
  Type 1:
 Given the kind and size of pipe and the flow rate          head loss ?

                                           Q    0.4
                                         V           2.037 m / s
                                           A 0.52 

                                                    4
      497 10 6        497 10 6
                                 1.006 10 6
     T  42.51.5 20  42.51.5
        0.5  2.037
  NR                  1.012 106
       1.006 10 6
  e     0.045
                 9 10 5
  D 0.5 103
  Moody f  0.013
     
                            2
                1000 2.037
  h f  0.013                5.5 m / km
                 0.5 2  9.81
Example 3
Cast iron pipe (e = 0.26), length = 2 km, diameter = 0.3m. Determine the
max. flow rate Q , If the allowable maximum head loss = 4.6m. T=10oC
 Type 2:
Given the kind and size of pipe and the head loss          flow rate ?
              2
        LV
 hF  f
        D 2g
        2000        V2
 4.6  f     
          0.3 2  9.81
              1
      0.0135
 V2          
                                 497 10 6       497 10 6
         f                                                   1.3110 6
                                T  42.51.5 10  42.51.5
                                    0.3  V
                             NR            6
                                                2.296 106 V  2
                                                                
                                  1.3110
                             e     0.26
                                            8.67 10 5  0.00009
                             D 0.3 103
Trial 1

 f  0.01 V  1.16 m/s
           eq1


 eq 2 N R  2.668 105
                                     V2 
                                              0.0135
                                                      1
                                                     
                                                 f
 e
    8.67 10  4
 D                                    N R  2.296 106V  2
                                                        
 Moody f  0.02
   

Trial 2

f  0.02 V  0.82 m/s
          eq1


eq 2 N R  1.886 105
 
e
   8.67 10  4
D
Moody f  0.021
  

            V= 0.82 m/s , Q = V*A = 0.058 m3/s
Example 3.5
Compute the discharge capacity of a 3-m diameter, wood stave
  pipe in its best condition carrying water at 10oC. It is allowed to
  have a head loss of 2m/km of pipe length.
  Type 2:
 Given the kind and size of pipe and the head loss               flow rate ?

 Solution 1:
           LV2              2ghf 1/ 2   1/ 2
                                     D 
    hf  f             V          
           D 2g              L    f  

          1000  V
                     2
                              0.12
     2 f              V 
                           2

          3  2(9.81)        f
    Table 3.1 : wood stave pipe: e = 0.18 – 0.9 mm, take e = 0.3 mm
     e 0.3
           0.0001
     D 3
                                                     VD   3V
   At T= 10oC,  = 1.31x10-6 m2/sec  N R                        2.29  106.V
                                                      1.31 106
• Solve by trial and error:
• Iteration 1:
                                 0.12
• Assume f = 0.02           V        V  2.45m / sec
                              2

                                 0.02
                            N R  2.29 106.2.45  5.6 106
From moody Diagram: f  0.0122

Iteration 2:
                                    0.12
update f = 0.0122          V2            V  3.14m / sec
                                   0.0122
                    N R  2.29 106.3.14  7.2 106
From moody Diagram: f  0.0121  0.0122


Iteration   f        V        NR
                                                       V2  3.15 m/s
  0         0.02     2.45    5.6106                                     32
   1        0.0122   3.14    7.2106        Solution: Q  VA  3.15.
   2        0.0121
                                                                          4
                            Convergence
                                                           22.27 m3 /s
Alternative Method for solution of Type 2 problems
                                                             1/ 2
                                           D3 / 2  2 gh f   
                              NR       f                   
                                             L  
                                                             
                                                             
  Type 2. Given the kind and size of pipe and the head loss                         flow rate ?

        Determines relative roughness e/D

        Given N R   f and e/D we can determine f (Moody diagram)
        Use Darcy-Weisbach to determine velocity and flow rate
 Because V is unknown we cannot calculate the Reynolds number
 However, if we know the friction loss hf, we can use the Darcy-Weisbach equation
 to write:                  LV2                   1/ 2   1/ 2
                                                              
                                                         2ghf         D
                      hf  f                       V           
                              D 2g                       L    f  

                          Re 
                               VD                             1   3 / 2  2ghf 1/ 2
                                                                    D                
 We also know that:                                  Re    1/ 2  
                                                                             
                                                                                     
                                                            f      L  
                                                                            
             2           1/ 2         
           D  2 gh f
             3/
                          
 NR f  
     1/ 2
                
                         
                                                 unknowns
             L                        
  Can be calculated based on                 Quantity plotted along the top of the Moody diagram
        available data
Moody Diagram                                D3 / 2  2 gh f                
                                                                                                                 1/ 2

                                                         N R f 1/ 2                                  
                                                                          L  
                                                                                                         
                                                                                                         




                                 Fully rough pipes
Resistance Coefficient f




                                                                                        Relative roughness e/D
                                Smooth pipes


                                               Reynolds number
Example 3.5
Compute the discharge capacity of a 3-m diameter, wood stave pipe in its best
  condition carrying water at 10oC. It is allowed to have a head loss of 2m/km
  of pipe length.

  Type 2: Given the kind and size of pipe and the head loss                      flow rate ?
  Solution 2:
   At T= 10oC,  = 1.31x10-6 m2/sec
                         1/ 2
            D  2 gh f
              3/ 2
                                    (3)3 2            2(9.81)(3)
   NR   f                                                      9.62  105
               L
               
                         
                                 1.31 106             1000
  Table 3.1 : wood pipe: e = 0.18 – 0.9 mm, take e = 0.3 mm e  0.3  0.0001
                                                            D 3
  From moody Diagram: f  0.0121

         LV   2
                    2 gh f 
                                     1/ 2
                                            D
                                                1/ 2
                                                                                            32
  hf  f      V  
                    L                     
                                             f        3.15m / sec , Q  VA  3.15.
         D 2g                                                                              4
                                                                              22.27 m3 /s
f = 0.0121
Example (type 2)
             1

                                                 H = 4 m, L = 200 m, and D = 0.05 m
                                         H
                                                 What is the discharge through the
                                     2           galvanized iron pipe?

                           L

Table : Galvanized iron pipe: e = 0.15 mm               e/D = 0.00015/0.05 = 0.003
    = 10-6 m2/s
We can write the energy equation between the water surface in the reservoir and the
free jet at the end of the pipe:       2              2
                                p1            V1  p        V
                                      h1        2  h2  2  hL
                                             2g          2g
                                                                   2
                                                   L V
                                             2
                                          V2
                               04000       f 
                                          2g       D  2g
                                                  2g 4        78.5
                                         V2             
                                                     L   1  4000 f
                                                1 f  
                                                     D
Example (continued)
Assume Initial value for f : fo = 0.026

                                                     78.5
Initial estimate for V:                V                        0.865 m/sec
                                               1  4000  0.026
                                                 DV
 Calculate the Reynolds number N R                      5  104  V  4.3  104
                                                  
 Updated the value of f from the Moody diagram f1                = 0.029
                           78.5
             V                        0.819 m/sec
                     1  4000  0.029
                     DV2
            NR               5  104  V  4.1 104
                     

Iteration     f          V              NR
0           0.026        0.865       4.3104                             V 2  0.814 m/s
1           0.029        0.819       4.1104              Solution:                           0.05 2
                                                                         Q VA  0.814 
2           0.0294       0.814       4.07104                                                     4
3           0.0294                              Convergence                  1.60  103 m3 /s
Initial estimate for f
A good initial estimate is to pick the f value that is valid for a fully rough pipe with
the specified relative roughness




  fo = 0.026                                                                      e/D = 0.003
Solution of Type 3 problems-uniform flow in a
                 single pipe

Given the kind of pipe, the head loss and flow rate          size of pipe ?
          Determines
    equivalent roughness e

                      Without D we cannot calculate the relative
      Problem?
                      roughness e/D, NR, or N R f

  Solution procedure: Iterate on f and D

 1.    Use the Darcy Weisbach equation and guess an initial value for f
 2.    Solve for D
 3.    Calculate e/D
 4.    Calculate NR
 5.    Update f
 6.    Solve for D
 7.    If new D different from old D go to step 3, otherwise done
Example (Type 3)
A pipeline is designed to carry crude oil (S = 0.93,  = 10-5 m2/s) with a discharge of 0.10
m3/s and a head loss per kilometer of 50 m. What diameter of steel pipe is needed?
Available pipe diameters are 20, 22, and 24 cm.
   From Table 3.1 : Steel pipe: e = 0.045 mm
   Darcy-Weisbach:
                                                    2
                                          Q
                                           
           L V
                 2
                                        L  A     L Q 4
                                                         2    2
                                                                    1 16 fLQ2
   hf  f                     hf  f         f               5
           D  2g                                 D  2g  D     D 2 g 2
                                                            2   4
                                        D 2g
                                                              1/ 5
                                     16  1000  0.102 
                        1/ 5
     16 fLQ       2

   D                            D                               f 1/ 5  0.440  f 1/ 5
                  
       2 g 2 h f                    2  9.81  2  50 
                                                          
     
   Make an initial guess for f :       fo = 0.015       D  0.440 0.0151/ 5  0.190 m
   Now we can calculate the relative roughness and the Reynolds number:
   e 0.045  103
                  0.00024                   
   D     D                                                                                     update f
          VD       Q D 4Q D 4Q 1                 1                                             f = 0.021
   NR                           12.7  103   66.8  103
                  A  D 2   D               D
Updated estimate for f




f1 = 0.021



                                e/D = 0.00024
Example Cont’d
          D  0.440  f 1/ 5
                               1                             Solution:
         N R  12.7  103 
                               D
                                                             D = 0.203
    From moody diagram, updated estimated for f :
                                                             Use next larger commercial
      f1 = 0.021              D = 0.203 m                    size:
                          N R  62.5 103         update f
                          e                                  D = 22 cm
                             0.00023
                          D

Iteration    f        D            NR       e/D
0           0.015     0.190        66.8103 0.00024
1           0.021     0.203        62.5103 0.00023
2           0.021                            Convergence
Example 3.6
Estimate the size of a uniform, horizontal welded-steel pipe installed to carry 14
   ft3/sec of water of 70oF (20oC). The allowable pressure loss is 17 ft/mi of
   pipe length.
  Solution 2:
  From Table : Steel pipe: ks = 0.046 mm
                          LV2                           Q 2
                                                          
   Darcy-Weisbach: hL  f                                
                          D 2g                        L  
                                                         A       L Q 2 42     1 16fLQ 2
                                               hL  f         f            
                    Q VA                             D 2g       D 2g  2D 4 D 5 2g 2
                                                                               1/ 5
                                                                  8 fLQ 2 
                                                             D 2 
                              1
                                 /5                               g hL 
        8  f  5280 14  2
     D                             f 1/ 5  4.33  f 1/ 5   a
          9.81  2 17 
  Let D = 2.5 ft, then          V = Q/A = 2.85 ft/sec
  Now by knowing the relative roughness and the Reynolds number:
    e 0.003
            0.0012
    D   2.5
                                             We get f =0.021
        VD  2.85 * 2.5
   NR                  6.6 *105
         1.08 *10   5
A better estimate of D can be obtained by substituting the latter
values into equation a, which gives


D  4.33  f 1/ 5  4.33 * 0.0211/ 5  2.0 ft

A new iteration provide
V = 4.46 ft/sec
NR = 8.3 x 105
e/D = 0.0015
f = 0.022, and
D = 2.0 ft.
More iterations will produce the same results.
Major losses formulas
• Several formulas have been developed in the past.
  Some of these formulas have faithfully been used in
  various hydraulic engineering practices.

     1. Darcy-Weisbach formula
     2. The Hazen -Williams Formula
     3. The Manning Formula
     4. The Chezy Formula
     5. The Strickler Formula



                                                   64
Empirical Formulas 1
• Hazen-Williams
 D  5cm    V  3.0m / sec
 V  1.318CHW Rh .63S 0.54
               0
                                             British Units


V  0.85CHW Rh
                              0.63           0.54
                                         S                SI Units
                                                                        D 2
                                                               wetted A       D
                            Rh  hydraulic Radius                       4 
            Simplified




                                                               wetted P   D  4
                                    hf
                            S
                                    L
                            C HW     Hazen Williams Coefficien t
            10.7 L
   hf        1.852
                           Q1.852                   SI Units
          CHW       D 4.87
CHW  Hazen Williams Coefficien t
CHW  Hazen Williams Coefficien t




                                    68
When V  3.0m / sec
                     0.081
             Vo 
 CH    C Ho    
             V 

Where:
CH = corrected value
CHo = value from table
Vo = velocity at CHo
V = actual velocity




                             69
Empirical Formulas 2

         Manning Formula

• This formula has extensively been used
  for open channel designs

• It is also quite commonly used for pipe
  flows

                                            70
• Manning

    1 2 / 3 1/ 2
 V  Rh S
    n               Rh  hydraulic Radius 
                                               wetted A D
                                                        
                                               wetted P   4
                         hf
                    S
   Simplified




                        L
                    n  Manning Coefficien t



     10.3 L nQ 
                2
hf                            SI Units
         D 5.33
                                                              71
1 2/ 3 1/ 2
                 V  Rh S
                    n
                                 Q2
                 h f  10.3n 2 L 16 / 3
                                D
                          L 2 2
               h f  6.35 1.33 n V
                         D

• n = Manning coefficient of roughness (See Table)
• Rh and S are as defined for Hazen-William
  formula.


                                                     72
73
74
Example
New Cast Iron (CHW = 130, n = 0.011) has length = 6 km and diameter = 30cm.
  Q= 0.32 m3/s, T=30o. Calculate the head loss due to friction using:

a) Hazen-William Method

           10.7 L
    hf      1.852
                          Q1.852
         CHW       D 4.87
             10.7  6000
     hf                     0.321.852  333m
            1301.852 0.34.87

b) Manning Method
          10.3 L nQ 
                       2
     hf 
              D 5.33
          10.3  6000 0.011 0.32
                                   2
     hf                5 .33
                                      470 m
                     0.3
Minor losses

It is due to the change
of the velocity of the
flowing fluid in the
magnitude        or   in
direction [turbulence
within bulk flow as it
moves through and
fitting]                   Flow pattern through a valve




                                                          76
• The minor losses occurs du to :
                • Valves
                • Tees
                • Bends
                • Reducers
                • Valves
                • And other appurtenances

• It has the common form

             V2       Q2
    hm  k L     kL
             2g      2 gA2
“minor” compared to friction losses in long pipelines but,

can be the dominant cause of head loss in shorter pipelines   77
Losses due to contraction
A sudden contraction in a pipe usually causes a marked drop in pressure
in the pipe due to both the increase in velocity and the loss of energy to
turbulence.
                         Along wall




                                                                   2
                                                              V2
  Along centerline                                    hc  kc
                                                              2g
Value of the coefficient Kc for sudden contraction


                                                     V2
Different pipe entrance




          increasing loss coefficient
Loss due to pipe entrance
General formula for head loss at the entrance of a pipe is also
expressed in term of velocity head of the pipe
                                         2
                                     V
                      hent  K ent
                                     2g




                                                                  81
Head Loss at the entrance of a Pipe
        (flow leaving a tank)



 Reentrant                              Sharp
(embeded)                               edge
 KL = 0.8                              KL = 0.5




 Slightly
 rounded
                                         Well
 KL = 0.2
                                       rounded
                                       KL = 0.04




                      V2
             hL  K L                              82
                      2g
Head Loss Due to a Sudden Contraction


         V 22
hL  K L
         2g

         2
        V2
hL 0.5
        2g



                                        83
Head losses due to pipe contraction may be greatly reduced by
introducing a gradual pipe transition known as a confusor




                          kc'

               2
           V2
 hc'  kc'
           2g
Head Loss Due to Gradual Contraction
        (reducer or nozzle)




                                       85
Losses due to Expansion
A sudden Expansion in a pipe




                                    (V1  V2 )   2
                               hE 
                                        2g
Note that the drop in the energy line is much
            larger than in the case of a contraction




                                     abrupt expansion




                          gradual expansion


smaller head loss than in the case of an abrupt expansion
Head Loss Due to a Sudden Enlargement


                V 12
       hL  K L
                2g
                         2
                 A1 
       K L  1     
                 A2 

or :




  hL 
            V1  V2    2


                2g

                                               88
Head losses due to pipe enlargement may be greatly reduced by
 introducing a gradual pipe transition known as a diffusor




           V  V2
             2       2
hE'  k E'  1
             2g
Head Loss Due to Gradual Enlargement
          (conical diffuser)




 hL K L
           V
            1    V2
                2      2
                           
                2g



       a           100        200    300    400

      KL            0.39       0.80   1.00   1.06




                                                    90
Gibson tests




               91
Another Typical values for various amount of rounding of
the lip




                                                           92
Head Loss at the Exit of a Pipe
                 (flow entering a tank)
                                 KL = 1.0              KL = 1.0




             V2
        hL 
             2g                                          KL = 1.0
                                  KL = 1.0




the entire kinetic energy of the exiting fluid (velocity V1) is
dissipated through viscous effects as the stream of fluid mixes
with the fluid in the tank and eventually comes to rest (V2 = 0).
                                                                93
Head Loss Due to Bends in Pipes



        V2
hb  kb
        2g




       R/D    1      2      4      6     10     16     20
       Kb    0.35   0.19   0.17   0.22   0.32   0.38   0.42


                                                              94
Miter bends
For situations in which space is limited,




                                            95
Head Loss Due to Pipe Fittings
(valves, elbows, bends, and tees)




                                                2
                                             V
                                    hv  K v
                                             2g




                                           96
97
The loss coefficient for elbows, bends, and tees




                                              98
Loss coefficients for pipe components (Table)
Minor loss coefficients (Table)
Minor loss calculation using equivalent
              pipe length

     kl D
Le 
       f
Energy and hydraulic grade lines




Unless local effects are of particular interests the changes in the EGL and HGL are
often shown as abrupt changes (even though the loss occurs over some distance)
Example
Given: Figure
Find: Estimate the elevation required in the
                                                                                                              b
    upper reservoir to produce a water
    discharge of 10 cfs in the system. What
    is the minimum pressure in the pipeline
    and what is the pressure there?
Solution:

                                                                    V2 p                   V2 p
                                                                  α1 1  1  z1   hL  αb b  b  zb
   V2 p                        V2 p                                 2g γ                   2g   γ
α1 1  1  z1   hL  α2 2  2  z2
   2g     γ                    2g γ                                                             Vb2 pb
                                                                        0  0  z1   hL  1 *        zb
0  0  z1   hL  0  0  z2                                                                  2g   γ
                             L V 2                                                 pb             Vb2              L V 2
 hL   K e  2 K b  K E  f                                                          z1  zb       Ke  Kb  f 
                             D  2g                                                 γ              2g               D  2g
                                                     L           430                                                                        2
K e  0.5; K b  0.4 (assumed); K E  1.0; f            0.025 *      10.75                                                    300  12.73
                                                                                         133  110.7  1.0  0.5  0.4  0.025     
                                                     D            1                                                              1  2 * 32.2
      Q     10                                                                           1.35 ft
V                 12.73 ft / s
      A  / 4 * 12                                                                   pb  62.4 * ( 1.53)  0.59 psig
                                                 2
z1  100  0.5  2 * 0.4  1.0  10.75
                                           12.73                                            VD   12.73 * 1
                                                     133 ft                         Re                    9 * 105
                                           2 * 32.2                                           1.14 * 10 5
Example
 In the figure shown two new cast iron pipes in series, D1 =0.6m ,
 D2 =0.4m length of the two pipes is 300m, level at A =80m , Q
 = 0.5m3/s (T=10oC).there are a sudden contraction between
 Pipe 1 and 2, and Sharp entrance at pipe 1.
 Fine the water level at B




  e = 0.26mm
v = 1.31×10-6
Q = 0.5 m3/s
Solution
Z A  ZB  hf
hL  h f 1  h f 2  hent  hc  hexit
                 2                2            2          2
        L1 V1       L2 V2         V1      V2         V22
hL  f1         f2         kent     kc     kexit
        D1 2 g      D2 2 g        2g      2g         2g
      Q      0.5                          Q         0.5
 V1                1.77 m/ sec , V2                    3.98 m/ sec ,
           π                                     π
      A1     0.62                       A2        0.42
           4                                      4
       VD                                VD
 Re1  1 1  8.1105 ,             Re 2  2 2  1.22 106 ,
         υ                                 υ
  0.26                                      
            0.00043,                            0.00065,
 D1 600                                      D1
 moody f1  0.017
                                    moody f 2  0.018
                                        


 hent  0.5,          hc  0.27,             hexit  1
2            2         2        2
        L1 V1       L2 V2         V1      V2         V22
hL  f1         f2         kent     kc     kexit
        D1 2 g      D2 2 g        2g      2g         2g


             300  1.77            300  3.98
                         2                      2
h f  0.017      .        0.018      .
             0.6  2 g             0.4  2 g
                            1.77 2      3.982   3.982 
                       0.5
                            2 g   0.27 2 g    2 g   13.36m
                                                       
                                                      

ZB = 80 – 13.36 = 66.64 m
Example
A pipe enlarge suddenly from D1=240mm to D2=480mm. the
H.G.L rises by 10 cm calculate the flow in the pipe
Solution   p1 V12        p2 V22
                  z1         z 2  hL
           rg 2 g        rg 2 g
           V12 V22         p2         p1        
                   hL  
                           rg  z 2    rg  z1 
                                                 
           2g 2g                                
           V12 V22 V1  V2
                 
                                             2

                                                    0.1
           2g 2g      2g

           V1 A1  V2 A2
           V1   
                
                    4
                           
                      0.24 2  V2      
                                       
                                           4 0.482   
           V1  4V2
           16V22 V22 4V2  V2
                    
                                                    
                                                     2

                                                          0.1
            2g    2g    2g
                    2
           6V2
                0.1
            2g
           V2  0.57 m / s  Q  V2 A2  0.57   0.482  0.103m 3 / s
                                                4
• Note that the above values are average
  typical values, actual values will depend
  on the make (manufacturer) of the
  components.
• See:
  – Catalogs
  – Hydraulic handbooks !!



                                              109

More Related Content

What's hot

Darcy´s law
Darcy´s lawDarcy´s law
Darcy´s lawoscar
 
Chapter 5 -momentum_equation_and_its_applications
Chapter 5 -momentum_equation_and_its_applicationsChapter 5 -momentum_equation_and_its_applications
Chapter 5 -momentum_equation_and_its_applicationsZackRule
 
Laminar Flow in pipes and Anuli Newtonian Fluids
Laminar Flow in pipes and Anuli Newtonian FluidsLaminar Flow in pipes and Anuli Newtonian Fluids
Laminar Flow in pipes and Anuli Newtonian FluidsUsman Shah
 
Stability of A Floating Body | Jameel Academy
Stability of  A Floating Body | Jameel AcademyStability of  A Floating Body | Jameel Academy
Stability of A Floating Body | Jameel AcademyJameel Academy
 
Fluid Flow Rate & Bernoulli’s Theorem Demonstration
Fluid Flow Rate & Bernoulli’s Theorem DemonstrationFluid Flow Rate & Bernoulli’s Theorem Demonstration
Fluid Flow Rate & Bernoulli’s Theorem DemonstrationRaboon Redar
 
Fluid MechanicsLosses in pipes dynamics of viscous flows
Fluid MechanicsLosses in pipes dynamics of viscous flowsFluid MechanicsLosses in pipes dynamics of viscous flows
Fluid MechanicsLosses in pipes dynamics of viscous flowsMohsin Siddique
 
Fluid Mechanic Lab - Hydrostatic Pressure
Fluid Mechanic Lab - Hydrostatic Pressure Fluid Mechanic Lab - Hydrostatic Pressure
Fluid Mechanic Lab - Hydrostatic Pressure MuhammadSRaniYah
 
Solution manual fundamentals of fluid mechanics (4th edition)
Solution manual   fundamentals of fluid mechanics (4th edition)Solution manual   fundamentals of fluid mechanics (4th edition)
Solution manual fundamentals of fluid mechanics (4th edition)Guilherme Gonçalves
 
Dymanics of open channel flow
Dymanics of open channel flowDymanics of open channel flow
Dymanics of open channel flowMohsin Siddique
 
8. fm 9 flow in pipes major loses co 3 copy
8. fm 9 flow in pipes  major loses co 3   copy8. fm 9 flow in pipes  major loses co 3   copy
8. fm 9 flow in pipes major loses co 3 copyZaza Eureka
 
Fluid Mechanics Chapter 3. Integral relations for a control volume
Fluid Mechanics Chapter 3. Integral relations for a control volumeFluid Mechanics Chapter 3. Integral relations for a control volume
Fluid Mechanics Chapter 3. Integral relations for a control volumeAddisu Dagne Zegeye
 
Fluid MechanicsVortex flow and impulse momentum
Fluid MechanicsVortex flow and impulse momentumFluid MechanicsVortex flow and impulse momentum
Fluid MechanicsVortex flow and impulse momentumMohsin Siddique
 
friction loss along a pipe
friction loss along a pipefriction loss along a pipe
friction loss along a pipeSaif al-din ali
 

What's hot (20)

Darcy´s law
Darcy´s lawDarcy´s law
Darcy´s law
 
Chapter 5 -momentum_equation_and_its_applications
Chapter 5 -momentum_equation_and_its_applicationsChapter 5 -momentum_equation_and_its_applications
Chapter 5 -momentum_equation_and_its_applications
 
Laminar Flow in pipes and Anuli Newtonian Fluids
Laminar Flow in pipes and Anuli Newtonian FluidsLaminar Flow in pipes and Anuli Newtonian Fluids
Laminar Flow in pipes and Anuli Newtonian Fluids
 
Stability of A Floating Body | Jameel Academy
Stability of  A Floating Body | Jameel AcademyStability of  A Floating Body | Jameel Academy
Stability of A Floating Body | Jameel Academy
 
Fluid Flow Rate & Bernoulli’s Theorem Demonstration
Fluid Flow Rate & Bernoulli’s Theorem DemonstrationFluid Flow Rate & Bernoulli’s Theorem Demonstration
Fluid Flow Rate & Bernoulli’s Theorem Demonstration
 
Fluid MechanicsLosses in pipes dynamics of viscous flows
Fluid MechanicsLosses in pipes dynamics of viscous flowsFluid MechanicsLosses in pipes dynamics of viscous flows
Fluid MechanicsLosses in pipes dynamics of viscous flows
 
Chap 03
Chap 03Chap 03
Chap 03
 
Fluid Mechanic Lab - Hydrostatic Pressure
Fluid Mechanic Lab - Hydrostatic Pressure Fluid Mechanic Lab - Hydrostatic Pressure
Fluid Mechanic Lab - Hydrostatic Pressure
 
Solution manual fundamentals of fluid mechanics (4th edition)
Solution manual   fundamentals of fluid mechanics (4th edition)Solution manual   fundamentals of fluid mechanics (4th edition)
Solution manual fundamentals of fluid mechanics (4th edition)
 
Fluid mechanics
Fluid mechanics Fluid mechanics
Fluid mechanics
 
Dymanics of open channel flow
Dymanics of open channel flowDymanics of open channel flow
Dymanics of open channel flow
 
8. fm 9 flow in pipes major loses co 3 copy
8. fm 9 flow in pipes  major loses co 3   copy8. fm 9 flow in pipes  major loses co 3   copy
8. fm 9 flow in pipes major loses co 3 copy
 
Flow measurement
Flow measurementFlow measurement
Flow measurement
 
Open Channel Flow
Open Channel FlowOpen Channel Flow
Open Channel Flow
 
Fluid Mechanics Chapter 3. Integral relations for a control volume
Fluid Mechanics Chapter 3. Integral relations for a control volumeFluid Mechanics Chapter 3. Integral relations for a control volume
Fluid Mechanics Chapter 3. Integral relations for a control volume
 
Fluid MechanicsVortex flow and impulse momentum
Fluid MechanicsVortex flow and impulse momentumFluid MechanicsVortex flow and impulse momentum
Fluid MechanicsVortex flow and impulse momentum
 
Fluid kinematics
Fluid kinematicsFluid kinematics
Fluid kinematics
 
Notches and weir
Notches and weirNotches and weir
Notches and weir
 
008
008008
008
 
friction loss along a pipe
friction loss along a pipefriction loss along a pipe
friction loss along a pipe
 

Similar to hydro chapter_3 by louy Al hami

Viscosity & flow
Viscosity & flowViscosity & flow
Viscosity & flowMidoOoz
 
Turbulent Flow 130120119126
Turbulent Flow 130120119126Turbulent Flow 130120119126
Turbulent Flow 130120119126Pandya Kartik
 
Terbulent Flow in fluid mechanics
Terbulent Flow in fluid mechanicsTerbulent Flow in fluid mechanics
Terbulent Flow in fluid mechanicsPavan Narkhede
 
Friction in noncircular conduits of fluid.pptx
Friction in noncircular conduits of fluid.pptxFriction in noncircular conduits of fluid.pptx
Friction in noncircular conduits of fluid.pptxMuhammadHabib533231
 
Friction losses in turbulent flow (Fanning Equation).pdf
Friction losses in turbulent flow (Fanning Equation).pdfFriction losses in turbulent flow (Fanning Equation).pdf
Friction losses in turbulent flow (Fanning Equation).pdfSharpmark256
 
Basic equation of fluid flow mechan.pptx
Basic equation of fluid flow mechan.pptxBasic equation of fluid flow mechan.pptx
Basic equation of fluid flow mechan.pptxAjithPArun1
 
Ch#1 ADVANCED OPEN CHANNEL HYDRAULICS.pdf
Ch#1 ADVANCED OPEN CHANNEL HYDRAULICS.pdfCh#1 ADVANCED OPEN CHANNEL HYDRAULICS.pdf
Ch#1 ADVANCED OPEN CHANNEL HYDRAULICS.pdfHadiqa Qadir
 
T1 - Essential Fluids - 2023.pptx
T1 - Essential Fluids - 2023.pptxT1 - Essential Fluids - 2023.pptx
T1 - Essential Fluids - 2023.pptxKeith Vaugh
 
Flow of viscous fluid through circular pipe
Flow of viscous fluid through circular pipeFlow of viscous fluid through circular pipe
Flow of viscous fluid through circular pipevaibhav tailor
 
Computational hydraulics
Computational hydraulicsComputational hydraulics
Computational hydraulicsRavi Teja
 

Similar to hydro chapter_3 by louy Al hami (20)

Chapter 1..ppt
Chapter 1..pptChapter 1..ppt
Chapter 1..ppt
 
Lecture 6
Lecture 6Lecture 6
Lecture 6
 
Presentation 2 ce 801
Presentation  2  ce 801Presentation  2  ce 801
Presentation 2 ce 801
 
Viscosity & flow
Viscosity & flowViscosity & flow
Viscosity & flow
 
Turbulence
TurbulenceTurbulence
Turbulence
 
Turbulent Flow 130120119126
Turbulent Flow 130120119126Turbulent Flow 130120119126
Turbulent Flow 130120119126
 
Terbulent Flow in fluid mechanics
Terbulent Flow in fluid mechanicsTerbulent Flow in fluid mechanics
Terbulent Flow in fluid mechanics
 
Fm final ppt
Fm final pptFm final ppt
Fm final ppt
 
Friction in noncircular conduits of fluid.pptx
Friction in noncircular conduits of fluid.pptxFriction in noncircular conduits of fluid.pptx
Friction in noncircular conduits of fluid.pptx
 
Friction losses in turbulent flow (Fanning Equation).pdf
Friction losses in turbulent flow (Fanning Equation).pdfFriction losses in turbulent flow (Fanning Equation).pdf
Friction losses in turbulent flow (Fanning Equation).pdf
 
Turbulent flow
Turbulent flowTurbulent flow
Turbulent flow
 
Flujo turbulento
Flujo turbulentoFlujo turbulento
Flujo turbulento
 
Basic equation of fluid flow mechan.pptx
Basic equation of fluid flow mechan.pptxBasic equation of fluid flow mechan.pptx
Basic equation of fluid flow mechan.pptx
 
Ch#1 ADVANCED OPEN CHANNEL HYDRAULICS.pdf
Ch#1 ADVANCED OPEN CHANNEL HYDRAULICS.pdfCh#1 ADVANCED OPEN CHANNEL HYDRAULICS.pdf
Ch#1 ADVANCED OPEN CHANNEL HYDRAULICS.pdf
 
T1 - Essential Fluids - 2023.pptx
T1 - Essential Fluids - 2023.pptxT1 - Essential Fluids - 2023.pptx
T1 - Essential Fluids - 2023.pptx
 
Steady Flow through Pipes
Steady Flow through PipesSteady Flow through Pipes
Steady Flow through Pipes
 
Qb103353
Qb103353Qb103353
Qb103353
 
Lecture notes 04
Lecture notes 04Lecture notes 04
Lecture notes 04
 
Flow of viscous fluid through circular pipe
Flow of viscous fluid through circular pipeFlow of viscous fluid through circular pipe
Flow of viscous fluid through circular pipe
 
Computational hydraulics
Computational hydraulicsComputational hydraulics
Computational hydraulics
 

More from Louy Alhamy

hydro chapter_11_hydrology_by louy al hami
hydro  chapter_11_hydrology_by louy al hami hydro  chapter_11_hydrology_by louy al hami
hydro chapter_11_hydrology_by louy al hami Louy Alhamy
 
hydro chapter_7_groundwater_by louy Al hami
hydro chapter_7_groundwater_by louy Al hami hydro chapter_7_groundwater_by louy Al hami
hydro chapter_7_groundwater_by louy Al hami Louy Alhamy
 
hydro chapter_5_a_by louy Al hami
hydro chapter_5_a_by louy Al hami hydro chapter_5_a_by louy Al hami
hydro chapter_5_a_by louy Al hami Louy Alhamy
 
hydro chapter_4_b_pipe_network_by louy Al hami
hydro  chapter_4_b_pipe_network_by louy Al hami hydro  chapter_4_b_pipe_network_by louy Al hami
hydro chapter_4_b_pipe_network_by louy Al hami Louy Alhamy
 
hydro chapter_4_a_by louy al hami
hydro chapter_4_a_by louy al hami hydro chapter_4_a_by louy al hami
hydro chapter_4_a_by louy al hami Louy Alhamy
 
economy Chapter7_by louy Al hami
economy Chapter7_by louy Al hamieconomy Chapter7_by louy Al hami
economy Chapter7_by louy Al hamiLouy Alhamy
 
economy Chapter6 2011_by louy Al hami
economy Chapter6 2011_by louy Al hami economy Chapter6 2011_by louy Al hami
economy Chapter6 2011_by louy Al hami Louy Alhamy
 
economy Chapter5_by louy Al hami
economy Chapter5_by louy Al hami economy Chapter5_by louy Al hami
economy Chapter5_by louy Al hami Louy Alhamy
 
economy Chapter4 part3_by louy al hami
economy Chapter4 part3_by louy al hami economy Chapter4 part3_by louy al hami
economy Chapter4 part3_by louy al hami Louy Alhamy
 
economy Ch4part2_by louy Al hami
economy Ch4part2_by louy Al hami economy Ch4part2_by louy Al hami
economy Ch4part2_by louy Al hami Louy Alhamy
 
economy Ch4 part.1 _by louy Alhami
economy Ch4 part.1 _by louy Alhamieconomy Ch4 part.1 _by louy Alhami
economy Ch4 part.1 _by louy AlhamiLouy Alhamy
 
economy Chapter2_by louy Al hami
economy Chapter2_by louy Al hami economy Chapter2_by louy Al hami
economy Chapter2_by louy Al hami Louy Alhamy
 

More from Louy Alhamy (12)

hydro chapter_11_hydrology_by louy al hami
hydro  chapter_11_hydrology_by louy al hami hydro  chapter_11_hydrology_by louy al hami
hydro chapter_11_hydrology_by louy al hami
 
hydro chapter_7_groundwater_by louy Al hami
hydro chapter_7_groundwater_by louy Al hami hydro chapter_7_groundwater_by louy Al hami
hydro chapter_7_groundwater_by louy Al hami
 
hydro chapter_5_a_by louy Al hami
hydro chapter_5_a_by louy Al hami hydro chapter_5_a_by louy Al hami
hydro chapter_5_a_by louy Al hami
 
hydro chapter_4_b_pipe_network_by louy Al hami
hydro  chapter_4_b_pipe_network_by louy Al hami hydro  chapter_4_b_pipe_network_by louy Al hami
hydro chapter_4_b_pipe_network_by louy Al hami
 
hydro chapter_4_a_by louy al hami
hydro chapter_4_a_by louy al hami hydro chapter_4_a_by louy al hami
hydro chapter_4_a_by louy al hami
 
economy Chapter7_by louy Al hami
economy Chapter7_by louy Al hamieconomy Chapter7_by louy Al hami
economy Chapter7_by louy Al hami
 
economy Chapter6 2011_by louy Al hami
economy Chapter6 2011_by louy Al hami economy Chapter6 2011_by louy Al hami
economy Chapter6 2011_by louy Al hami
 
economy Chapter5_by louy Al hami
economy Chapter5_by louy Al hami economy Chapter5_by louy Al hami
economy Chapter5_by louy Al hami
 
economy Chapter4 part3_by louy al hami
economy Chapter4 part3_by louy al hami economy Chapter4 part3_by louy al hami
economy Chapter4 part3_by louy al hami
 
economy Ch4part2_by louy Al hami
economy Ch4part2_by louy Al hami economy Ch4part2_by louy Al hami
economy Ch4part2_by louy Al hami
 
economy Ch4 part.1 _by louy Alhami
economy Ch4 part.1 _by louy Alhamieconomy Ch4 part.1 _by louy Alhami
economy Ch4 part.1 _by louy Alhami
 
economy Chapter2_by louy Al hami
economy Chapter2_by louy Al hami economy Chapter2_by louy Al hami
economy Chapter2_by louy Al hami
 

Recently uploaded

ISYU TUNGKOL SA SEKSWLADIDA (ISSUE ABOUT SEXUALITY
ISYU TUNGKOL SA SEKSWLADIDA (ISSUE ABOUT SEXUALITYISYU TUNGKOL SA SEKSWLADIDA (ISSUE ABOUT SEXUALITY
ISYU TUNGKOL SA SEKSWLADIDA (ISSUE ABOUT SEXUALITYKayeClaireEstoconing
 
Inclusivity Essentials_ Creating Accessible Websites for Nonprofits .pdf
Inclusivity Essentials_ Creating Accessible Websites for Nonprofits .pdfInclusivity Essentials_ Creating Accessible Websites for Nonprofits .pdf
Inclusivity Essentials_ Creating Accessible Websites for Nonprofits .pdfTechSoup
 
Grade 9 Quarter 4 Dll Grade 9 Quarter 4 DLL.pdf
Grade 9 Quarter 4 Dll Grade 9 Quarter 4 DLL.pdfGrade 9 Quarter 4 Dll Grade 9 Quarter 4 DLL.pdf
Grade 9 Quarter 4 Dll Grade 9 Quarter 4 DLL.pdfJemuel Francisco
 
THEORIES OF ORGANIZATION-PUBLIC ADMINISTRATION
THEORIES OF ORGANIZATION-PUBLIC ADMINISTRATIONTHEORIES OF ORGANIZATION-PUBLIC ADMINISTRATION
THEORIES OF ORGANIZATION-PUBLIC ADMINISTRATIONHumphrey A Beña
 
Choosing the Right CBSE School A Comprehensive Guide for Parents
Choosing the Right CBSE School A Comprehensive Guide for ParentsChoosing the Right CBSE School A Comprehensive Guide for Parents
Choosing the Right CBSE School A Comprehensive Guide for Parentsnavabharathschool99
 
Global Lehigh Strategic Initiatives (without descriptions)
Global Lehigh Strategic Initiatives (without descriptions)Global Lehigh Strategic Initiatives (without descriptions)
Global Lehigh Strategic Initiatives (without descriptions)cama23
 
Integumentary System SMP B. Pharm Sem I.ppt
Integumentary System SMP B. Pharm Sem I.pptIntegumentary System SMP B. Pharm Sem I.ppt
Integumentary System SMP B. Pharm Sem I.pptshraddhaparab530
 
4.16.24 21st Century Movements for Black Lives.pptx
4.16.24 21st Century Movements for Black Lives.pptx4.16.24 21st Century Movements for Black Lives.pptx
4.16.24 21st Century Movements for Black Lives.pptxmary850239
 
Concurrency Control in Database Management system
Concurrency Control in Database Management systemConcurrency Control in Database Management system
Concurrency Control in Database Management systemChristalin Nelson
 
Q4-PPT-Music9_Lesson-1-Romantic-Opera.pptx
Q4-PPT-Music9_Lesson-1-Romantic-Opera.pptxQ4-PPT-Music9_Lesson-1-Romantic-Opera.pptx
Q4-PPT-Music9_Lesson-1-Romantic-Opera.pptxlancelewisportillo
 
Field Attribute Index Feature in Odoo 17
Field Attribute Index Feature in Odoo 17Field Attribute Index Feature in Odoo 17
Field Attribute Index Feature in Odoo 17Celine George
 
ANG SEKTOR NG agrikultura.pptx QUARTER 4
ANG SEKTOR NG agrikultura.pptx QUARTER 4ANG SEKTOR NG agrikultura.pptx QUARTER 4
ANG SEKTOR NG agrikultura.pptx QUARTER 4MiaBumagat1
 
Visit to a blind student's school🧑‍🦯🧑‍🦯(community medicine)
Visit to a blind student's school🧑‍🦯🧑‍🦯(community medicine)Visit to a blind student's school🧑‍🦯🧑‍🦯(community medicine)
Visit to a blind student's school🧑‍🦯🧑‍🦯(community medicine)lakshayb543
 
Student Profile Sample - We help schools to connect the data they have, with ...
Student Profile Sample - We help schools to connect the data they have, with ...Student Profile Sample - We help schools to connect the data they have, with ...
Student Profile Sample - We help schools to connect the data they have, with ...Seán Kennedy
 
Earth Day Presentation wow hello nice great
Earth Day Presentation wow hello nice greatEarth Day Presentation wow hello nice great
Earth Day Presentation wow hello nice greatYousafMalik24
 
ICS2208 Lecture6 Notes for SL spaces.pdf
ICS2208 Lecture6 Notes for SL spaces.pdfICS2208 Lecture6 Notes for SL spaces.pdf
ICS2208 Lecture6 Notes for SL spaces.pdfVanessa Camilleri
 
4.18.24 Movement Legacies, Reflection, and Review.pptx
4.18.24 Movement Legacies, Reflection, and Review.pptx4.18.24 Movement Legacies, Reflection, and Review.pptx
4.18.24 Movement Legacies, Reflection, and Review.pptxmary850239
 
What is Model Inheritance in Odoo 17 ERP
What is Model Inheritance in Odoo 17 ERPWhat is Model Inheritance in Odoo 17 ERP
What is Model Inheritance in Odoo 17 ERPCeline George
 
Activity 2-unit 2-update 2024. English translation
Activity 2-unit 2-update 2024. English translationActivity 2-unit 2-update 2024. English translation
Activity 2-unit 2-update 2024. English translationRosabel UA
 

Recently uploaded (20)

ISYU TUNGKOL SA SEKSWLADIDA (ISSUE ABOUT SEXUALITY
ISYU TUNGKOL SA SEKSWLADIDA (ISSUE ABOUT SEXUALITYISYU TUNGKOL SA SEKSWLADIDA (ISSUE ABOUT SEXUALITY
ISYU TUNGKOL SA SEKSWLADIDA (ISSUE ABOUT SEXUALITY
 
Inclusivity Essentials_ Creating Accessible Websites for Nonprofits .pdf
Inclusivity Essentials_ Creating Accessible Websites for Nonprofits .pdfInclusivity Essentials_ Creating Accessible Websites for Nonprofits .pdf
Inclusivity Essentials_ Creating Accessible Websites for Nonprofits .pdf
 
Grade 9 Quarter 4 Dll Grade 9 Quarter 4 DLL.pdf
Grade 9 Quarter 4 Dll Grade 9 Quarter 4 DLL.pdfGrade 9 Quarter 4 Dll Grade 9 Quarter 4 DLL.pdf
Grade 9 Quarter 4 Dll Grade 9 Quarter 4 DLL.pdf
 
THEORIES OF ORGANIZATION-PUBLIC ADMINISTRATION
THEORIES OF ORGANIZATION-PUBLIC ADMINISTRATIONTHEORIES OF ORGANIZATION-PUBLIC ADMINISTRATION
THEORIES OF ORGANIZATION-PUBLIC ADMINISTRATION
 
Choosing the Right CBSE School A Comprehensive Guide for Parents
Choosing the Right CBSE School A Comprehensive Guide for ParentsChoosing the Right CBSE School A Comprehensive Guide for Parents
Choosing the Right CBSE School A Comprehensive Guide for Parents
 
Global Lehigh Strategic Initiatives (without descriptions)
Global Lehigh Strategic Initiatives (without descriptions)Global Lehigh Strategic Initiatives (without descriptions)
Global Lehigh Strategic Initiatives (without descriptions)
 
Integumentary System SMP B. Pharm Sem I.ppt
Integumentary System SMP B. Pharm Sem I.pptIntegumentary System SMP B. Pharm Sem I.ppt
Integumentary System SMP B. Pharm Sem I.ppt
 
4.16.24 21st Century Movements for Black Lives.pptx
4.16.24 21st Century Movements for Black Lives.pptx4.16.24 21st Century Movements for Black Lives.pptx
4.16.24 21st Century Movements for Black Lives.pptx
 
Concurrency Control in Database Management system
Concurrency Control in Database Management systemConcurrency Control in Database Management system
Concurrency Control in Database Management system
 
Q4-PPT-Music9_Lesson-1-Romantic-Opera.pptx
Q4-PPT-Music9_Lesson-1-Romantic-Opera.pptxQ4-PPT-Music9_Lesson-1-Romantic-Opera.pptx
Q4-PPT-Music9_Lesson-1-Romantic-Opera.pptx
 
Field Attribute Index Feature in Odoo 17
Field Attribute Index Feature in Odoo 17Field Attribute Index Feature in Odoo 17
Field Attribute Index Feature in Odoo 17
 
ANG SEKTOR NG agrikultura.pptx QUARTER 4
ANG SEKTOR NG agrikultura.pptx QUARTER 4ANG SEKTOR NG agrikultura.pptx QUARTER 4
ANG SEKTOR NG agrikultura.pptx QUARTER 4
 
Visit to a blind student's school🧑‍🦯🧑‍🦯(community medicine)
Visit to a blind student's school🧑‍🦯🧑‍🦯(community medicine)Visit to a blind student's school🧑‍🦯🧑‍🦯(community medicine)
Visit to a blind student's school🧑‍🦯🧑‍🦯(community medicine)
 
Student Profile Sample - We help schools to connect the data they have, with ...
Student Profile Sample - We help schools to connect the data they have, with ...Student Profile Sample - We help schools to connect the data they have, with ...
Student Profile Sample - We help schools to connect the data they have, with ...
 
LEFT_ON_C'N_ PRELIMS_EL_DORADO_2024.pptx
LEFT_ON_C'N_ PRELIMS_EL_DORADO_2024.pptxLEFT_ON_C'N_ PRELIMS_EL_DORADO_2024.pptx
LEFT_ON_C'N_ PRELIMS_EL_DORADO_2024.pptx
 
Earth Day Presentation wow hello nice great
Earth Day Presentation wow hello nice greatEarth Day Presentation wow hello nice great
Earth Day Presentation wow hello nice great
 
ICS2208 Lecture6 Notes for SL spaces.pdf
ICS2208 Lecture6 Notes for SL spaces.pdfICS2208 Lecture6 Notes for SL spaces.pdf
ICS2208 Lecture6 Notes for SL spaces.pdf
 
4.18.24 Movement Legacies, Reflection, and Review.pptx
4.18.24 Movement Legacies, Reflection, and Review.pptx4.18.24 Movement Legacies, Reflection, and Review.pptx
4.18.24 Movement Legacies, Reflection, and Review.pptx
 
What is Model Inheritance in Odoo 17 ERP
What is Model Inheritance in Odoo 17 ERPWhat is Model Inheritance in Odoo 17 ERP
What is Model Inheritance in Odoo 17 ERP
 
Activity 2-unit 2-update 2024. English translation
Activity 2-unit 2-update 2024. English translationActivity 2-unit 2-update 2024. English translation
Activity 2-unit 2-update 2024. English translation
 

hydro chapter_3 by louy Al hami

  • 1. Civil Engineering Department Hydraulics and Hydrology – CE 352 Prof. Majed Abu-Zreig Chapter 3 Water Flow in Pipes
  • 2. 3.1 Description of A Pipe Flow • Water pipes in our homes and the distribution system • Pipes carry hydraulic fluid to various components of vehicles and machines • Natural systems of “pipes” that carry blood throughout our body and air into and out of our lungs. 2
  • 3. • Pipe Flow: refers to a full water flow in a closed conduits or circular cross section under a certain pressure gradient. • The pipe flow at any cross section can be described by:  cross section (A),  elevation (h), measured with respect to a horizontal reference datum.  pressure (P), varies from one point to another, for a given cross section variation is neglected  The flow velocity (v), v = Q/A. 3
  • 4. Difference between open-channel flow and the pipe flow Pipe flow Open-channel flow • The pipe is completely filled • Water flows without with the fluid being transported. completely filling the pipe. • The main driving force is likely • Gravity alone is the to be a pressure gradient along driving force, the water the pipe. flows down a hill. 4
  • 5. Types of Flow  Steady and Unsteady flow The flow parameters such as velocity (v), pressure (P) and density (r) of a fluid flow are independent of time in a steady flow. In unsteady flow they are independent. For a steady flow v t x ,y ,z o o o 0 For an unsteady flow v t x ,y ,z o o o 0 If the variations in any fluid’s parameters are small, the average is constant, then the fluid is considered to be steady 5
  • 6.  Uniform and non-uniform flow A flow is uniform if the flow characteristics at any given instant remain the same at different points in the direction of flow, otherwise it is termed as non-uniform flow. For a uniform flow v s t o 0 For a non-uniform flow v s t o 0 6
  • 7. Examples:  The flow through a long uniform pipe diameter at a constant rate is steady uniform flow.  The flow through a long uniform pipe diameter at a varying rate is unsteady uniform flow.  The flow through a diverging pipe diameter at a constant rate is a steady non-uniform flow.  The flow through a diverging pipe diameter at a varying rate is an unsteady non-uniform flow. 7
  • 8.  Laminar and turbulent flow Laminar flow: The fluid particles move along smooth well defined path or streamlines that are parallel, thus particles move in laminas or layers, smoothly gliding over each other. Turbulent flow: The fluid particles do not move in orderly manner and they occupy different relative positions in successive cross-sections. There is a small fluctuation in magnitude and direction of the velocity of the fluid particles transitional flow The flow occurs between laminar and turbulent flow 8
  • 9. 3.2 Reynolds Experiment Reynolds performed a very carefully prepared pipe flow experiment. 9
  • 10. Increasing flow velocity 10
  • 11. Reynolds Experiment • Reynold found that transition from laminar to turbulent flow in a pipe depends not only on the velocity, but only on the pipe diameter and the viscosity of the fluid. • This relationship between these variables is commonly known as Reynolds number (NR) VDr VD Inertial Forces NR      Viscous Forces It can be shown that the Reynolds number is a measure of the ratio of the inertial forces to the viscous forces in the flow FI  ma FV   A 11
  • 12. Reynolds number rVD VD NR     where V: mean velocity in the pipe [L/T] D: pipe diameter [L] r: density of flowing fluid [M/L3] : dynamic viscosity [M/LT] : kinematic viscosity [L2/T] 12
  • 13. 13
  • 14. It has been found by many experiments that for flows in circular pipes, the critical Reynolds number is about 2000 Flow laminar when NR < Critical NR Flow turbulent when NR > Critical NR The transition from laminar to turbulent flow does not always happened at NR = 2000 but varies due to experiments conditions….….this known as transitional range 14
  • 15. Laminar Vs. Turbulent flows Laminar flows characterized Turbulent flows characterized by: by • low velocities • high velocities • small length scales • large length scales • high kinematic viscosities • low kinematic viscosities • NR < Critical NR • NR > Critical NR • Viscous forces are • Inertial forces are dominant. dominant 15
  • 16. Example 3.1 40 mm diameter circular pipe carries water at 20oC. Calculate the largest flow rate (Q) which laminar flow can be expected. D  0.04m   1106 at T  20o C VD V (0.04) NR   2000   2000  V  0.05m / sec  110 6  Q  V . A  0.05  (0.04) 2  6.28 105 m3 / sec 4 16
  • 17. 3.3 Forces in Pipe Flow • Cross section and elevation of the pipe are varied along the axial direction of the flow. 17
  • 18. For Incompressible and Steady flows: Conservation law of mass r.dVol11'  r.dVol22'  mass flux ( fluid mass) Mass enters the Mass leaves the control volume control volume dVol11' dVol 2 2' r.  r. dt dt dS1 dS 2 r . A1  r . A2  r . A1.V1  r . A2 .V2  r .Q dt dt Continuity equation for Incompressible Steady flow A1.V1  A2 .V2  Q 18
  • 19. Apply Newton’s Second Law:     dV M V 2 M V1   F  M a  M dt  t F x  P A1  P2 A2  Fx  Wx 1 Fx is the axial direction force exerted on the control volume by the wall of the pipe. but M t  r .Q  mass flow rate  Fx  r.Q(Vx2  Vx1 )   F r.Q(V  2  V 1) F y  r .Q(V y2  V y1 ) F Conservation of z  r .Q(Vz2  Vz1 ) moment equation 19
  • 20. Example 3.2 dA= 40 mm, dB= 20 mm, PA= 500,000 N/m2, Q=0.01m3/sec. Determine the reaction force at the hinge. 20
  • 21. 3.4 Energy Head in Pipe Flow Water flow in pipes may contain energy in three basic forms: 1- Kinetic energy, 2- potential energy, 3- pressure energy. 21
  • 22. Consider the control volume: • In time interval dt: - Water particles at sec.1-1 move to sec. 1`-1` with velocity V1. - Water particles at sec.2-2 move to sec. 2`-2` with velocity V2. • To satisfy continuity equation: A1.V1.dt  A2 .V2 .dt • The work done by the pressure force P . A1.ds1  P . A1.V1.dt 1 1 ……. on section 1-1  P2 . A2 .ds2   P2 . A2 .V2 .dt ……. on section 2-2 -ve sign because P2 is in the opposite direction to distance traveled ds2 22
  • 23. • The work done by the gravity force : Work  W .h  mg.h m  r .Volume  A1 L  A1V1dt • The kinetic energy: Work  rg. A1.V1dt.(h1  h2 ) 1 1 1 M .V2  M .V1  r. A1.V1.dt (V22  V1 ) 2 2 2 2 2 2 The total work done by all forces is equal to the change in kinetic energy: 1 P .Q.dt  P2 .Q.dt  rg.Q.dt.(h1  h2 )  r.Q.dt (V22  V1 ) 2 1 2 Dividing both sides by rgQdt 2 2 Bernoulli Equation V1 P V P  1  h1  2  2  h2 2g  2g  Energy per unit weight of water 23 OR: Energy Head
  • 24. Energy head and Head loss in pipe flow 24
  • 25. 2 V2 P2 H2    h2 2g  Energy = Kinetic + Pressure + Elevation head head head head 2 V P H1  1  1  h1 2g  Notice that: • In reality, certain amount of energy loss (hL) occurs when the water mass flow from one section to another. • The energy relationship between two sections can be written as: 2 2 V1 P V2 P2   h1  1   h2  hL 2g  2g  25
  • 26. Example 3.4 The tank is being drained through 12 in pipe. The discharge = 3200 gpm, The 26 Total head loss = 11.5 ft. find the h?
  • 27. Example In the figure shown: Where the discharge through the system is 0.05 m3/s, the total losses through the pipe is 10 v2/2g where v is the velocity of water in 0.15 m diameter pipe, the water in the final outlet exposed to atmosphere.
  • 28. Example In the figure shown: Where the discharge through the system is 0.05 m3/s, the total losses through the pipe is 10 v2/2g where v is the velocity of water in 0.15 m diameter pipe, the water in the final outlet exposed to atmosphere. Calculate the required height (h =?) below the tank Calculate the required height (h =?) below the tank 0.05 V Q   2.83m / s 4 0.15 A  2 0.05 V Q   6.366m / s 4 0.10  A  2 p1 V12 p2 V22   z1    z 2  hL rg 2 g rg 2 g 0  0  (h  5)  0  6.3662  20  102.832 2 * 9.81 2 * 9.81 h  21.147 m
  • 29. Without calculation sketch the (E.G.L) and (H.G.L)
  • 30. Basic components of a typical pipe system 30
  • 31. Calculation of Head (Energy) Losses: In General: When a fluid is flowing through a pipe, the fluid experiences some resistance due to which some of energy (head) of fluid is lost. Energy Losses (Head losses) Major Losses Minor losses loss of head due to pipe Loss due to the change of friction and to viscous the velocity of the flowing dissipation in flowing fluid in the magnitude or in water direction as it moves through fitting like Valves, Tees, Bends and Reducers. 31
  • 32. 3.5 Losses of Head due to Friction • Energy loss through friction in the length of pipeline is commonly termed the major loss hf • This is the loss of head due to pipe friction and to the viscous dissipation in flowing water. • Several studies have been found the resistance to flow in a pipe is: - Independent of pressure under which the water flows - Linearly proportional to the pipe length, L - Inversely proportional to some water power of the pipe diameter D - Proportional to some power of the mean velocity, V - Related to the roughness of the pipe, if the flow is turbulent
  • 33. The resistance to flow in a pipe is a function of: • The pipe length, L • The pipe diameter, D • The mean velocity, V • The properties of the fluid () • The roughness of the pipe, (the flow is turbulent). 33
  • 34. Darcy-Weisbach Equation 2 2 Where: L V 8f LQ hL  f   f is the friction factor D 2 g g D5  2 L is pipe length D is pipe diameter Q is the flow rate hL is the loss due to friction It is conveniently expressed in terms of velocity (kinetic) head in the pipe The friction factor is function of different terms:  e  rVD e   VD e  f  F NR ,   F ,   F   D ,   D     D Renold number Relative roughness
  • 35. Friction Factor: (f) • For Laminar flow: (NR < 2000) [depends only on Reynolds’ number and not on the surface roughness] 64 f  NR • For turbulent flow in smooth pipes (e/D = 0) with 4000 < NR < 105 is 0.316 f  1/ 4 NR 35
  • 36. Friction Factor f The thickness of the laminar sublayer  decrease with an increase in NR laminar flow f independent of relative NR < 2000 Smooth roughness e/D  e  '  1.7e 64 1 N f  f   2 log 10  R  2.51   pipe wall NR f    f varies with NR and e/D  e   transitionally     2 log 10     rough 1 D 2.51   e f  3.7 N R f    pipe wall 0.08e   '  1.7e   Colebrook formula  turbulent flow f independent of NR rough NR > 4000 e 1  D    0.08e '  2 log 10  3.7  f  e pipe wall
  • 37. Moody diagram • A convenient chart was prepared by Lewis F. Moody and commonly called the Moody diagram of friction factors for pipe flow, There are 4 zones of pipe flow in the chart: • A laminar flow zone where f is simple linear function of NR • A critical zone (shaded) where values are uncertain because the flow might be neither laminar nor truly turbulent • A transition zone where f is a function of both NR and relative roughness • A zone of fully developed turbulence where the value of f depends solely on the relative roughness and independent of the Reynolds Number
  • 38. 38
  • 39. Laminar Marks Reynolds Number independence
  • 40. Typical values of the absolute roughness (e) are given in table 3.1 40
  • 41. Notes: Alternative to Moody Diagram • Swamee-Jain Equation (1976) 0.25 f  2   e 5.74  log10  3.7 D  Re 0.9     Explicit expression 10-6<e/D,10-2; 5000<NR<108 41
  • 42. Problems (head loss) Three types of problems for uniform flow in a single pipe:  Type 1: Given the kind and size of pipe and the flow rate head loss ?  Type 2: Given the kind and size of pipe and the head loss flow rate ?  Type 3: Given the kind of pipe, the head loss and flow rate size of pipe ?
  • 43. Solving Turbulent flow Problems There are 3 types of problems • Given: L, D, V solve for hf – Compute ks/D, Re then f from moody diagram then find hf and V • Given: hf, L, D solve for V 3/ 2 D 2 gh – Compute ks/D the value  L Then calculate f, V f and Q • Given: Q, L, hf solve for D, Iterative solution – Iterative process Assume f, calculate V and Re, check f from moody diagram – Assume new f, calculate V and Re, then check f
  • 44. Moody Diagram  D3 / 2  2 gh f  1/ 2 N R f 1/ 2       L    Fully rough pipes Resistance Coefficient f Relative roughness e/D Smooth pipes Reynolds number
  • 45. Example 1 The water flow in Asphalted cast Iron pipe (e = 0.12mm) has a diameter 20cm at 20oC. Is 0.05 m3/s. determine the losses due to friction per 1 km  Type 1: Given the kind and size of pipe and the flow rate head loss ? 0.05m 3 /s V  1.59m/s  π/4  0.2 m 2 2  T  20o C  υ  1.0110 6 m 2 /s e  0.12mm e 0.12mm   0.0006 Moody f = 0.018 D 200mm VD 1.59  0.2 NR    314852  3.15 105  1.0110 6 L V2  1,000 m  1.59  2 hf  f  0.018   D 2g    0.20 m  2 9.81 m/s 2    45  11.55 m
  • 46. Example 2 The water flow in commercial steel pipe (e = 0.045mm) has a diameter 0.5m at 20oC. Q=0.4 m3/s. determine the losses due to friction per 1 km  Type 1: Given the kind and size of pipe and the flow rate head loss ? Q 0.4 V   2.037 m / s A 0.52  4 497 10 6 497 10 6    1.006 10 6 T  42.51.5 20  42.51.5 0.5  2.037 NR   1.012 106 1.006 10 6 e 0.045   9 10 5 D 0.5 103 Moody f  0.013   2 1000 2.037 h f  0.013    5.5 m / km 0.5 2  9.81
  • 47. Example 3 Cast iron pipe (e = 0.26), length = 2 km, diameter = 0.3m. Determine the max. flow rate Q , If the allowable maximum head loss = 4.6m. T=10oC  Type 2: Given the kind and size of pipe and the head loss flow rate ? 2 LV hF  f D 2g 2000 V2 4.6  f  0.3 2  9.81  1 0.0135 V2   497 10 6 497 10 6 f    1.3110 6 T  42.51.5 10  42.51.5 0.3  V NR  6  2.296 106 V  2  1.3110 e 0.26   8.67 10 5  0.00009 D 0.3 103
  • 48. Trial 1 f  0.01 V  1.16 m/s eq1 eq 2 N R  2.668 105  V2  0.0135  1  f e  8.67 10  4 D N R  2.296 106V  2  Moody f  0.02  Trial 2 f  0.02 V  0.82 m/s eq1 eq 2 N R  1.886 105  e  8.67 10  4 D Moody f  0.021  V= 0.82 m/s , Q = V*A = 0.058 m3/s
  • 49. Example 3.5 Compute the discharge capacity of a 3-m diameter, wood stave pipe in its best condition carrying water at 10oC. It is allowed to have a head loss of 2m/km of pipe length.  Type 2: Given the kind and size of pipe and the head loss flow rate ? Solution 1: LV2  2ghf 1/ 2   1/ 2  D  hf  f V      D 2g   L    f    1000  V 2 0.12 2 f  V  2  3  2(9.81) f Table 3.1 : wood stave pipe: e = 0.18 – 0.9 mm, take e = 0.3 mm e 0.3   0.0001 D 3 VD 3V At T= 10oC,  = 1.31x10-6 m2/sec  N R    2.29  106.V  1.31 106
  • 50. • Solve by trial and error: • Iteration 1: 0.12 • Assume f = 0.02  V   V  2.45m / sec 2 0.02 N R  2.29 106.2.45  5.6 106 From moody Diagram: f  0.0122 Iteration 2: 0.12 update f = 0.0122  V2   V  3.14m / sec 0.0122 N R  2.29 106.3.14  7.2 106 From moody Diagram: f  0.0121  0.0122 Iteration f V NR V2  3.15 m/s 0 0.02 2.45 5.6106   32 1 0.0122 3.14 7.2106 Solution: Q  VA  3.15. 2 0.0121 4 Convergence  22.27 m3 /s
  • 51. Alternative Method for solution of Type 2 problems 1/ 2 D3 / 2  2 gh f  NR f      L    Type 2. Given the kind and size of pipe and the head loss flow rate ? Determines relative roughness e/D Given N R f and e/D we can determine f (Moody diagram) Use Darcy-Weisbach to determine velocity and flow rate Because V is unknown we cannot calculate the Reynolds number However, if we know the friction loss hf, we can use the Darcy-Weisbach equation to write: LV2  1/ 2   1/ 2   2ghf D hf  f V      D 2g   L    f   Re  VD   1   3 / 2  2ghf 1/ 2 D   We also know that: Re    1/ 2          f      L      2 1/ 2   D  2 gh f 3/  NR f   1/ 2     unknowns    L   Can be calculated based on Quantity plotted along the top of the Moody diagram available data
  • 52. Moody Diagram  D3 / 2  2 gh f  1/ 2 N R f 1/ 2       L    Fully rough pipes Resistance Coefficient f Relative roughness e/D Smooth pipes Reynolds number
  • 53. Example 3.5 Compute the discharge capacity of a 3-m diameter, wood stave pipe in its best condition carrying water at 10oC. It is allowed to have a head loss of 2m/km of pipe length. Type 2: Given the kind and size of pipe and the head loss flow rate ? Solution 2: At T= 10oC,  = 1.31x10-6 m2/sec 1/ 2 D  2 gh f 3/ 2  (3)3 2 2(9.81)(3) NR f      9.62  105   L    1.31 106 1000 Table 3.1 : wood pipe: e = 0.18 – 0.9 mm, take e = 0.3 mm e  0.3  0.0001 D 3 From moody Diagram: f  0.0121 LV 2  2 gh f  1/ 2 D 1/ 2   32 hf  f V    L      f   3.15m / sec , Q  VA  3.15. D 2g     4  22.27 m3 /s
  • 55. Example (type 2) 1 H = 4 m, L = 200 m, and D = 0.05 m H What is the discharge through the 2 galvanized iron pipe? L Table : Galvanized iron pipe: e = 0.15 mm e/D = 0.00015/0.05 = 0.003  = 10-6 m2/s We can write the energy equation between the water surface in the reservoir and the free jet at the end of the pipe: 2 2 p1 V1 p V  h1   2  h2  2  hL  2g  2g 2  L V 2 V2 04000  f  2g  D  2g 2g 4 78.5 V2   L 1  4000 f 1 f   D
  • 56. Example (continued) Assume Initial value for f : fo = 0.026 78.5 Initial estimate for V: V  0.865 m/sec 1  4000  0.026 DV Calculate the Reynolds number N R   5  104  V  4.3  104  Updated the value of f from the Moody diagram f1 = 0.029 78.5 V  0.819 m/sec 1  4000  0.029 DV2 NR   5  104  V  4.1 104  Iteration f V NR 0 0.026 0.865 4.3104 V 2  0.814 m/s 1 0.029 0.819 4.1104 Solution:   0.05 2 Q VA  0.814  2 0.0294 0.814 4.07104 4 3 0.0294 Convergence  1.60  103 m3 /s
  • 57. Initial estimate for f A good initial estimate is to pick the f value that is valid for a fully rough pipe with the specified relative roughness fo = 0.026 e/D = 0.003
  • 58. Solution of Type 3 problems-uniform flow in a single pipe Given the kind of pipe, the head loss and flow rate size of pipe ? Determines equivalent roughness e Without D we cannot calculate the relative Problem? roughness e/D, NR, or N R f Solution procedure: Iterate on f and D 1. Use the Darcy Weisbach equation and guess an initial value for f 2. Solve for D 3. Calculate e/D 4. Calculate NR 5. Update f 6. Solve for D 7. If new D different from old D go to step 3, otherwise done
  • 59. Example (Type 3) A pipeline is designed to carry crude oil (S = 0.93,  = 10-5 m2/s) with a discharge of 0.10 m3/s and a head loss per kilometer of 50 m. What diameter of steel pipe is needed? Available pipe diameters are 20, 22, and 24 cm. From Table 3.1 : Steel pipe: e = 0.045 mm Darcy-Weisbach: 2 Q    L V 2 L  A  L Q 4 2 2 1 16 fLQ2 hf  f    hf  f  f   5  D  2g  D  2g  D D 2 g 2 2 4 D 2g 1/ 5 16  1000  0.102  1/ 5 16 fLQ  2 D  D f 1/ 5  0.440  f 1/ 5  2 g 2 h f   2  9.81  2  50    Make an initial guess for f : fo = 0.015 D  0.440 0.0151/ 5  0.190 m Now we can calculate the relative roughness and the Reynolds number: e 0.045  103   0.00024  D D update f VD Q D 4Q D 4Q 1 1 f = 0.021 NR      12.7  103   66.8  103  A  D 2   D D
  • 60. Updated estimate for f f1 = 0.021 e/D = 0.00024
  • 61. Example Cont’d D  0.440  f 1/ 5 1 Solution: N R  12.7  103  D D = 0.203 From moody diagram, updated estimated for f : Use next larger commercial f1 = 0.021 D = 0.203 m size: N R  62.5 103 update f e D = 22 cm  0.00023 D Iteration f D NR e/D 0 0.015 0.190 66.8103 0.00024 1 0.021 0.203 62.5103 0.00023 2 0.021 Convergence
  • 62. Example 3.6 Estimate the size of a uniform, horizontal welded-steel pipe installed to carry 14 ft3/sec of water of 70oF (20oC). The allowable pressure loss is 17 ft/mi of pipe length. Solution 2: From Table : Steel pipe: ks = 0.046 mm LV2 Q 2   Darcy-Weisbach: hL  f   D 2g L   A L Q 2 42 1 16fLQ 2 hL  f f  Q VA D 2g D 2g  2D 4 D 5 2g 2 1/ 5  8 fLQ 2   D 2   1 /5  g hL   8  f  5280 14  2 D  f 1/ 5  4.33  f 1/ 5 a  9.81  2 17  Let D = 2.5 ft, then V = Q/A = 2.85 ft/sec Now by knowing the relative roughness and the Reynolds number: e 0.003   0.0012 D 2.5 We get f =0.021 VD 2.85 * 2.5 NR    6.6 *105  1.08 *10 5
  • 63. A better estimate of D can be obtained by substituting the latter values into equation a, which gives D  4.33  f 1/ 5  4.33 * 0.0211/ 5  2.0 ft A new iteration provide V = 4.46 ft/sec NR = 8.3 x 105 e/D = 0.0015 f = 0.022, and D = 2.0 ft. More iterations will produce the same results.
  • 64. Major losses formulas • Several formulas have been developed in the past. Some of these formulas have faithfully been used in various hydraulic engineering practices. 1. Darcy-Weisbach formula 2. The Hazen -Williams Formula 3. The Manning Formula 4. The Chezy Formula 5. The Strickler Formula 64
  • 65. Empirical Formulas 1 • Hazen-Williams D  5cm    V  3.0m / sec V  1.318CHW Rh .63S 0.54 0 British Units V  0.85CHW Rh 0.63 0.54 S SI Units D 2 wetted A D Rh  hydraulic Radius   4  Simplified wetted P D 4 hf S L C HW  Hazen Williams Coefficien t 10.7 L hf  1.852 Q1.852 SI Units CHW D 4.87
  • 66.
  • 67. CHW  Hazen Williams Coefficien t
  • 68. CHW  Hazen Williams Coefficien t 68
  • 69. When V  3.0m / sec 0.081 Vo  CH  C Ho   V  Where: CH = corrected value CHo = value from table Vo = velocity at CHo V = actual velocity 69
  • 70. Empirical Formulas 2 Manning Formula • This formula has extensively been used for open channel designs • It is also quite commonly used for pipe flows 70
  • 71. • Manning 1 2 / 3 1/ 2 V  Rh S n Rh  hydraulic Radius  wetted A D  wetted P 4 hf S Simplified L n  Manning Coefficien t 10.3 L nQ  2 hf  SI Units D 5.33 71
  • 72. 1 2/ 3 1/ 2 V  Rh S n Q2 h f  10.3n 2 L 16 / 3 D L 2 2 h f  6.35 1.33 n V D • n = Manning coefficient of roughness (See Table) • Rh and S are as defined for Hazen-William formula. 72
  • 73. 73
  • 74. 74
  • 75. Example New Cast Iron (CHW = 130, n = 0.011) has length = 6 km and diameter = 30cm. Q= 0.32 m3/s, T=30o. Calculate the head loss due to friction using: a) Hazen-William Method 10.7 L hf  1.852 Q1.852 CHW D 4.87 10.7  6000 hf  0.321.852  333m 1301.852 0.34.87 b) Manning Method 10.3 L nQ  2 hf  D 5.33 10.3  6000 0.011 0.32 2 hf  5 .33  470 m 0.3
  • 76. Minor losses It is due to the change of the velocity of the flowing fluid in the magnitude or in direction [turbulence within bulk flow as it moves through and fitting] Flow pattern through a valve 76
  • 77. • The minor losses occurs du to : • Valves • Tees • Bends • Reducers • Valves • And other appurtenances • It has the common form V2 Q2 hm  k L  kL 2g 2 gA2 “minor” compared to friction losses in long pipelines but, can be the dominant cause of head loss in shorter pipelines 77
  • 78. Losses due to contraction A sudden contraction in a pipe usually causes a marked drop in pressure in the pipe due to both the increase in velocity and the loss of energy to turbulence. Along wall 2 V2 Along centerline hc  kc 2g
  • 79. Value of the coefficient Kc for sudden contraction V2
  • 80. Different pipe entrance increasing loss coefficient
  • 81. Loss due to pipe entrance General formula for head loss at the entrance of a pipe is also expressed in term of velocity head of the pipe 2 V hent  K ent 2g 81
  • 82. Head Loss at the entrance of a Pipe (flow leaving a tank) Reentrant Sharp (embeded) edge KL = 0.8 KL = 0.5 Slightly rounded Well KL = 0.2 rounded KL = 0.04 V2 hL  K L 82 2g
  • 83. Head Loss Due to a Sudden Contraction V 22 hL  K L 2g 2 V2 hL 0.5 2g 83
  • 84. Head losses due to pipe contraction may be greatly reduced by introducing a gradual pipe transition known as a confusor kc' 2 V2 hc'  kc' 2g
  • 85. Head Loss Due to Gradual Contraction (reducer or nozzle) 85
  • 86. Losses due to Expansion A sudden Expansion in a pipe (V1  V2 ) 2 hE  2g
  • 87. Note that the drop in the energy line is much larger than in the case of a contraction abrupt expansion gradual expansion smaller head loss than in the case of an abrupt expansion
  • 88. Head Loss Due to a Sudden Enlargement V 12 hL  K L 2g 2  A1  K L  1    A2  or : hL  V1  V2  2 2g 88
  • 89. Head losses due to pipe enlargement may be greatly reduced by introducing a gradual pipe transition known as a diffusor V  V2 2 2 hE'  k E' 1 2g
  • 90. Head Loss Due to Gradual Enlargement (conical diffuser) hL K L V 1  V2 2 2  2g a 100 200 300 400 KL 0.39 0.80 1.00 1.06 90
  • 92. Another Typical values for various amount of rounding of the lip 92
  • 93. Head Loss at the Exit of a Pipe (flow entering a tank) KL = 1.0 KL = 1.0 V2 hL  2g KL = 1.0 KL = 1.0 the entire kinetic energy of the exiting fluid (velocity V1) is dissipated through viscous effects as the stream of fluid mixes with the fluid in the tank and eventually comes to rest (V2 = 0). 93
  • 94. Head Loss Due to Bends in Pipes V2 hb  kb 2g R/D 1 2 4 6 10 16 20 Kb 0.35 0.19 0.17 0.22 0.32 0.38 0.42 94
  • 95. Miter bends For situations in which space is limited, 95
  • 96. Head Loss Due to Pipe Fittings (valves, elbows, bends, and tees) 2 V hv  K v 2g 96
  • 97. 97
  • 98. The loss coefficient for elbows, bends, and tees 98
  • 99. Loss coefficients for pipe components (Table)
  • 101. Minor loss calculation using equivalent pipe length kl D Le  f
  • 102. Energy and hydraulic grade lines Unless local effects are of particular interests the changes in the EGL and HGL are often shown as abrupt changes (even though the loss occurs over some distance)
  • 103. Example Given: Figure Find: Estimate the elevation required in the b upper reservoir to produce a water discharge of 10 cfs in the system. What is the minimum pressure in the pipeline and what is the pressure there? Solution: V2 p V2 p α1 1  1  z1   hL  αb b  b  zb V2 p V2 p 2g γ 2g γ α1 1  1  z1   hL  α2 2  2  z2 2g γ 2g γ Vb2 pb 0  0  z1   hL  1 *   zb 0  0  z1   hL  0  0  z2 2g γ  L V 2 pb Vb2  L V 2  hL   K e  2 K b  K E  f   z1  zb    Ke  Kb  f   D  2g γ 2g  D  2g L 430 2 K e  0.5; K b  0.4 (assumed); K E  1.0; f  0.025 *  10.75  300  12.73  133  110.7  1.0  0.5  0.4  0.025  D 1  1  2 * 32.2 Q 10  1.35 ft V    12.73 ft / s A  / 4 * 12 pb  62.4 * ( 1.53)  0.59 psig 2 z1  100  0.5  2 * 0.4  1.0  10.75 12.73 VD 12.73 * 1  133 ft Re    9 * 105 2 * 32.2  1.14 * 10 5
  • 104. Example In the figure shown two new cast iron pipes in series, D1 =0.6m , D2 =0.4m length of the two pipes is 300m, level at A =80m , Q = 0.5m3/s (T=10oC).there are a sudden contraction between Pipe 1 and 2, and Sharp entrance at pipe 1. Fine the water level at B e = 0.26mm v = 1.31×10-6 Q = 0.5 m3/s
  • 105. Solution Z A  ZB  hf hL  h f 1  h f 2  hent  hc  hexit 2 2 2 2 L1 V1 L2 V2 V1 V2 V22 hL  f1  f2  kent  kc  kexit D1 2 g D2 2 g 2g 2g 2g Q 0.5 Q 0.5 V1    1.77 m/ sec , V2    3.98 m/ sec , π π A1 0.62 A2 0.42 4 4 VD VD Re1  1 1  8.1105 , Re 2  2 2  1.22 106 , υ υ  0.26    0.00043,  0.00065, D1 600 D1 moody f1  0.017   moody f 2  0.018   hent  0.5, hc  0.27, hexit  1
  • 106. 2 2 2 2 L1 V1 L2 V2 V1 V2 V22 hL  f1  f2  kent  kc  kexit D1 2 g D2 2 g 2g 2g 2g  300  1.77  300  3.98 2 2 h f  0.017  .  0.018  .  0.6  2 g  0.4  2 g  1.77 2   3.982   3.982   0.5  2 g   0.27 2 g    2 g   13.36m            ZB = 80 – 13.36 = 66.64 m
  • 107. Example A pipe enlarge suddenly from D1=240mm to D2=480mm. the H.G.L rises by 10 cm calculate the flow in the pipe
  • 108. Solution p1 V12 p2 V22   z1    z 2  hL rg 2 g rg 2 g V12 V22  p2   p1    hL    rg  z 2    rg  z1     2g 2g     V12 V22 V1  V2    2  0.1 2g 2g 2g V1 A1  V2 A2 V1   4  0.24 2  V2   4 0.482  V1  4V2 16V22 V22 4V2  V2     2  0.1 2g 2g 2g 2 6V2  0.1 2g V2  0.57 m / s  Q  V2 A2  0.57   0.482  0.103m 3 / s 4
  • 109. • Note that the above values are average typical values, actual values will depend on the make (manufacturer) of the components. • See: – Catalogs – Hydraulic handbooks !! 109